Difference between revisions of "2021 Fall AMC 10B Problems/Problem 11"

(Solution)
(Solution)
Line 8: Line 8:
 
Thus, we have that <math>C-H=H-A</math>, or <math>A=2H-C</math>.  
 
Thus, we have that <math>C-H=H-A</math>, or <math>A=2H-C</math>.  
 
By some formulas, <math>C=\pi{r}^2=\pi</math> and <math>H=6\cdot\frac12\cdot1\cdot(\frac12\cdot\sqrt3)=\frac{3\sqrt3}2</math>.  
 
By some formulas, <math>C=\pi{r}^2=\pi</math> and <math>H=6\cdot\frac12\cdot1\cdot(\frac12\cdot\sqrt3)=\frac{3\sqrt3}2</math>.  
Thus, <math>A=3\sqrt3-\pi</math> or <math>\boxed{(\textbf{B}.)}.</math>.
+
Thus, <math>A=3\sqrt3-\pi</math> or <math>\boxed{(\textbf{B})}</math>.
 +
 
 +
~Hefei417, or 陆畅 Sunny from China
  
 
==See Also==
 
==See Also==
 
{{AMC10 box|year=2021 Fall|ab=B|num-a=12|num-b=10}}
 
{{AMC10 box|year=2021 Fall|ab=B|num-a=12|num-b=10}}
 
{{MAA Notice}}
 
{{MAA Notice}}

Revision as of 09:57, 23 November 2021

Problem

I hope someone will enter this.


Solution

Let the hexagon described be of area $H$ and let the circle's area be $C$. Let the area we want to aim for be $A$. Thus, we have that $C-H=H-A$, or $A=2H-C$. By some formulas, $C=\pi{r}^2=\pi$ and $H=6\cdot\frac12\cdot1\cdot(\frac12\cdot\sqrt3)=\frac{3\sqrt3}2$. Thus, $A=3\sqrt3-\pi$ or $\boxed{(\textbf{B})}$.

~Hefei417, or 陆畅 Sunny from China

See Also

2021 Fall AMC 10B (ProblemsAnswer KeyResources)
Preceded by
Problem 10
Followed by
Problem 12
1 2 3 4 5 6 7 8 9 10 11 12 13 14 15 16 17 18 19 20 21 22 23 24 25
All AMC 10 Problems and Solutions

The problems on this page are copyrighted by the Mathematical Association of America's American Mathematics Competitions. AMC logo.png